Resuelve cada uno de los problemas de valor inicial, usando el método de la transformada de Laplace

Transformada de Laplace

Resuelve cada uno de los problemas de valor inicial, usando el método de la transformada de Laplace

1. Y''-5y'+4y=e 2t ;    y(0)=1, y'(0)=-1
2. Y''+y= t sent  ;       y(0)=1, y'(0)=2

1 Respuesta

Respuesta
1

y '' - 5 y ' + 4 y = e^(2t);  aplicamos la transformada a ambos lados:

L{y ''} - 5 L {y '} + 4 L{y} = L {e^2t};

s^2 L{y} - s*y(0) - y'(0) - 5 [sL{y} - y(0)] + 4 L{y} = 1 / (s-2);  

Ya no tengo derivadas;  reemplazo con los valores iniciales:

s^2 L{y} - s +1 - 5 [sL{y} - 1] + 4 L{y} = 1 / (s-2) 

s^2 L{y} - s +1 - 5sL{y} +5 + 4 L{y} = 1 / (s-2) 

(s^2 -5s +4) *L{y} - s + 6 = 1 / (s-2);

(s^2 -5s +4) *L{y} = [1 / (s-2)] + s - 6;  despejo L{y}:

L{y} = {1 / [(s-2)*(s^2 -5s +4)]} + [s/(s^2 -5s +4)]  - [6/(s^2 -5s +4)]; despejo y:

y=L^(-1){1 / [(s-2)*(s^2 -5s +4)]} + L^(-1)[s/(s^2 -5s +4)] - L^(-1)[6/(s^2 -5s +4)];

Como (s^2 -5s +4) es factorizable:  5+-√(25 - 16) / 2;  (5+-3)/2;  s=1; s=4:

Por espacio, hago las fracciones parciales de cada inversa por separado:

1°) 1 / [(s-2)*(s-1)*(s-4)] = A/(s-2) + B/(s-1) + C/(s-4);

1/[(s-2)*(s-1)*(s-4)] = [2)*(s-1)*(s-4)]

1 = A(s-1)(s-4) + B(s-2)(s-4) + C(s-2)(s-1);  doy valores a s:

1= A*1*(-2) + 0 B + 0C;   para s=2;  A=(-1/2);  

1 = 0A + B*(-1)(-3) + 0C;  para s=1;  B=(1/3);

1 = 0A + 0B + C*2*3;  para s=4;  C=1/6;

L^(-1) { (-1/2)* [1/(s-2)]  + (1/3)*[1/(s-1)] + (1/6)*[1/(s-4)]};

Como la Transformada inversa de 1/(s-a) = e^(at):

## (-1/2)e^2t + (1/3)e^t + (1/6)e^4t

2°)  s/[(s-1)(s-4)];  evitaré pasos intermedios:  

A/(s-1) + B/(s-4);   s= A(s-4) + B(s-1);

1 = A(-3)+0B;  para s=1;  A=(-1/3)

4= 0A +3B;  para s=4;  B=4/3;

(-1/3)[1/(s-1)] + (4/3)[1/(s-4)];  Transformada inversa:

## (-1/3)e^t + (4/3)e^4t;

3°) [6/(s^2 -5s +4)] o:  6/[(s-4)(s-1)];

6=A/(s-4) + B/(s-1);  6 = A(s-1) + B(s-4);

6=3A;  para s=4;  A=2;

6= -3B;  para s=1;  B=-2;

2*[1/(s-4)] - 2*[1/(s-1)];  Transformada inversa:

## 2e^4t - 2e^t;  

Todo junto ahora, con y(t);  y(0)=1;  y '(0) =(-1):

y(t) = (-1/2)e^2t + (1/3)e^t + (1/6)e^4t +(-1/3)e^t + (4/3)e^4t +2e^4t - 2e^t;

####  y(t) =(21/6)e^4t - (1/2)e^2t -2e^t

Luego intento el segundo.

Y''+y= t sent  ;       y(0)=1, y'(0)=2;   a la derecha, por propiedad de derivada.

s^2 L{y} - s*y(0) - y'(0) + L{y} = -F ' L{sent};

s^2 L{y} - s - 2 + L{y} = -F ' [1/(s^2+1)];

(s^2+1) L{y} - s - 2 = -  [2s/(s^2+1)^2];

(s^2+1) L{y} = s+2 -  [2s/(s^2+1)^2];

L{y} = [(s+2)/(s^2+1)]  -  [2s/(s^2+1)^3];

y = L^(-1) {[(s+2)/(s^2+1)]  -  [2s/(s^2+1)^3]);  

y = L^(-1) [(s+2)/(s^2+1)]  -   L^(-1) [2s/(s^2+1)^3]);  o:

y = L^(-1) [(s/(s^2+1)] + 2*L^(-1) [(1/(s^2+1)]   - 2*L^(-1) [s/(s^2+1)^3]);

Las dos primeras son directas:  cost + 2sent; 

La segunda es deducible de la propiedad de la derivada:  recordar:

a) L { t sent} =  F '[1/(s^2+1)] = (-1)^1 * (-2s)/(s^2+1)^2;  2s/(s^2+1)^2

b) L {t^2 cost} = (-1)^2 F '' [s/(s^2+1)] = 2s(s^2-3) / (s^2+1)^3;

Resto ambos:  [2s(s^2+1) - 2s(s^2-3)] / (s^2+1)^3;

2s*4 / (s^2+1)^3 = 8s / (s^2+1)^3;  

pero como tengo:  s/(s^2+1)^3;  divido ambos miembros por 8 y queda:

(1/8)* { L{t sent} - L{t^2cost}] = s/(s^2+1)^3;  FINALMENTE:

y = cost + 2sent - 2 (tsent - t^2cost);  o:

y = (2t^2 + 1) cost + 2(1-t)sent.

Corrijo el final del último (olvidé el 1/8):

y = cost + 2sent - 2*(1/8) (tsent - t^2cost);  o:

y = cost + 2sent - (1/4) (tsent - t^2cost);  o:

y = cost* [1+ (t^2/4)] + sent [2-(t/4)].

Añade tu respuesta

Haz clic para o

Más respuestas relacionadas